Question 71

If $$\left(\frac{\tan \theta - \sec \theta + 1}{\tan \theta + \sec \theta - 1}\right) \sec \theta = \frac{1}{k}$$, then k = 


Create a FREE account and get:

  • Free SSC Study Material - 18000 Questions
  • 230+ SSC previous papers with solutions PDF
  • 100+ SSC Online Tests for Free

cracku

Boost your Prep!

Download App